Functional Analysis, Show that the range of a bounded linear operator

In summary, the range of a bounded linear operator T: X \rightarrow Y is not necessarily closed. My problem is that I can't find a limit point of R(T) that doesn't belong to R(T).
  • #1
Eduardo
2
0

Homework Statement


Show that the range [tex]\mathcal{R}(T)[/tex] of a bounded linear operator [tex]T: X \rightarrow Y[/tex] is not necessarily closed.
Hint: Use the linear bounded operator [tex]T: l^{\infty} \rightarrow l^{\infty}[/tex] defined by [tex](\eta_{j}) = T x, \eta_{j} = \xi_{j}/j, x = (\xi_{j})[/tex].

Homework Equations





The Attempt at a Solution


My idea was to find an element [tex]y \in l^{\infty}[/tex] that does not belong to the range and then try to build a convergent sequence in [tex]\mathcal{R}(T)[/tex] that has limit [tex] y [/tex]. The element [tex]y = (1, 1, \ldots)[/tex] satisfy the criteria because [tex]T^{-1}y = \{ x\}[/tex], with [tex]x = (\xi_{j}), \xi_{j} = j[/tex], but, clearly, [tex]x \not\in l^{\infty}[/tex], therefore, [tex]y \not\in \mathcal{R}(T)[/tex]. The problem arise when I try to build the sequence, because [tex](T x_{m})[/tex] with [tex]x_{m} \in l^{\infty}[/tex] cannot converge to [tex] y [/tex]. Briefly, my problem is that I can´t find a limit point of [tex]\mathcal{R}(T)[/tex] that doesn´t belong to [tex]\mathcal{R}(T)[/tex].
 
Physics news on Phys.org
  • #2
Think from the other direction. Since x needs to be bounded, the elements in Tx must decrease to zero eventually. Which means that any element that's on the boundary of the image set must also have its elements converge to zero. Can you find a sequence nk that converges to zero, such that knk is unbounded?
 
  • #3
Just, logged into thank you, Office Shredder, for your quick and helpful answer. Wich was very nice because it was precisely what I needed, to unblock my fixed ideas o a change of approach to face the problem and let me think or find an answer instead of just give me the result inmediately. Thanks again.
 
  • #4
who did you find the find a limit point of
R(T)
that doesn´t belong to
R(T)?
 

Related to Functional Analysis, Show that the range of a bounded linear operator

1. What is functional analysis?

Functional analysis is a branch of mathematics that deals with the study of vector spaces and linear operators, particularly in infinite-dimensional spaces. It is an important tool in many areas of mathematics and has applications in physics, engineering, and economics.

2. What does it mean for a linear operator to be bounded?

A bounded linear operator is one that maps elements from one vector space to another in a way that preserves the distances between vectors. This means that the operator does not stretch or shrink vectors by an infinite amount, ensuring that the operator is well-behaved and has a finite limit.

3. How do you show that the range of a bounded linear operator is a closed subspace?

To show that the range of a bounded linear operator is a closed subspace, we must first prove that it is a subspace. This can be done by showing that it contains the zero vector, is closed under addition and scalar multiplication, and is non-empty. Then, to prove that it is closed, we must show that for any convergent sequence in the range, the limit also exists in the range. This can be done using the boundedness of the operator and the continuity of the operator.

4. What is the importance of studying the range of a bounded linear operator?

The range of a bounded linear operator is important because it provides information about the behavior and properties of the operator. It can help determine the invertibility of the operator, its dimension, and its kernel (null space). It also plays a crucial role in the study of functional analysis, as it allows us to analyze the behavior of operators in infinite-dimensional spaces.

5. Can the range of a bounded linear operator ever be the entire vector space it maps to?

No, the range of a bounded linear operator can never be the entire vector space it maps to. This is because a bounded linear operator cannot have an infinite limit, which would be necessary for the range to cover the entire vector space. However, the range can be dense in the vector space, meaning that any element in the vector space can be approximated by elements in the range.

Similar threads

  • Calculus and Beyond Homework Help
Replies
2
Views
721
  • Calculus and Beyond Homework Help
2
Replies
43
Views
3K
  • Calculus and Beyond Homework Help
Replies
14
Views
542
  • Calculus and Beyond Homework Help
Replies
1
Views
477
  • Calculus and Beyond Homework Help
Replies
8
Views
661
  • Calculus and Beyond Homework Help
Replies
2
Views
865
  • Calculus and Beyond Homework Help
Replies
9
Views
1K
  • Calculus and Beyond Homework Help
Replies
15
Views
1K
  • Calculus and Beyond Homework Help
Replies
1
Views
326
  • Calculus and Beyond Homework Help
Replies
5
Views
1K
Back
Top